PT 50, LR 2 (Sec. 4), Q. 6: WHY am I not getting this? Forum

Prepare for the LSAT or discuss it with others in this forum.
Post Reply
User avatar
RaleighStClair

Bronze
Posts: 481
Joined: Tue May 24, 2011 12:10 am

PT 50, LR 2 (Sec. 4), Q. 6: WHY am I not getting this?

Post by RaleighStClair » Fri Sep 09, 2011 12:17 pm

Of course the hardest problem for me on this section is in the first 10 questions. Apparently everyone else found it to be easy, as there's virtually nothing on the interwebz about it.

For question 6, I just do not understand how M's claim suggests E's premise could be used to support a conflicting conclusion. What is the conflicting conclusion? Is it that democracy is NOT the best form of government? (LMK if I'm including too many specifics in this - not sure of the protocol).

I chose D, as an unsure guessish response, fwiw.

User avatar
timmydoeslsat

Bronze
Posts: 148
Joined: Wed Aug 03, 2011 2:07 pm

Re: PT 50, LR 2 (Sec. 4), Q. 6: WHY am I not getting this?

Post by timmydoeslsat » Fri Sep 09, 2011 12:30 pm

Elena states that the best form of government is one that fosters the belief in its citizens that they actually have a say in how the government operates. She then concludes that democracy is the best form of government.

Marsha then tells Elena that there are many situations other than just democracy in which that belief is fostered. So her statement suggests that democracy is not the only one that fits that bill. According to what Elena stated, we would then accept the idea that all of those governments that facilitate this belief are in fact the best form of government.

Please tell me if you would like me to explain this more.

User avatar
RaleighStClair

Bronze
Posts: 481
Joined: Tue May 24, 2011 12:10 am

Re: PT 50, LR 2 (Sec. 4), Q. 6: WHY am I not getting this?

Post by RaleighStClair » Fri Sep 09, 2011 12:34 pm

OHHHHH. Haha ok thank you very much! The conflicting conclusion is that there are more governments than just democracies that could qualify Elena's premise.

User avatar
timmydoeslsat

Bronze
Posts: 148
Joined: Wed Aug 03, 2011 2:07 pm

Re: PT 50, LR 2 (Sec. 4), Q. 6: WHY am I not getting this?

Post by timmydoeslsat » Fri Sep 09, 2011 1:10 pm

Exactly. Notice that we do not know that Marsha would say "Democracy is not the best form."

Marsha is saying that there are other candidates that fit the bill described in the statement Elena gave.

Want to continue reading?

Register now to search topics and post comments!

Absolutely FREE!


Post Reply

Return to “LSAT Prep and Discussion Forum”